[Math] Some case when the central limit theorem fails

probability theory

If I understand correctly, for various versions of the central limit theorems (CLT), when applying to a sequence of random variables, each random variable is required to have finite mean and finite variance, plus some other conditions depending on the version of the CLT.

Months ago, I heard of something, probably about some case when the classical CLT fails, which I haven't been able to understand. I am not sure if my following description is correct, but that is perhaps the best I can recall:

if one random variable in the sequence dominates (in some sense, such as in terms of magnitude?) the other random
variables, then the central limit theorem doesn't hold.

I was wondering if someone is able to figure out what the quote is trying to say?

Thanks and regards!

PS: A paper named Asymptotic Distribution Theory for the Kalman Filter State Estimator was mentioned regarding the above quote. I don't quite understand the paper, so cannot figure out how it helps to clarify the quote. But I guess Section "3.2 Remarks on Theorems and Corollaries" on page "1999" might be related.

Best Answer

There are various ways in which the CLT can "fail", depending on which hypotheses are violated. Here's one. Suppose $X_k$ are independent random variables with $E[X_k] = \mu_k$ and variances $\sigma_k^2$, and let $s_n^2 = \sum_{k=1}^n \sigma_k^2$ and $S_n = \sum_{k=1}^n (X_k - \mu_k)$. Suppose also that $\max_{k \le n} \sigma_k/s_n \to 0$ as $n \to \infty$ (so in that sense no $X_k$ is "dominant" in $S_n$). Then Lindeberg's condition is both necessary and sufficient for $S_n/s_n$ to converge in distribution to ${\mathscr N}(0,1)$.

EDIT: Here's a nice example where the Central Limit Theorem fails. Let $X_n$ be independent with $P(X_n = 2^n) = P(X_n = -2^n) = 2^{-2n-1}$, $P(X_n = 0) = 1 - 2^{-2n}$. Thus $E[X_n] = 0$ and $\sigma_n = 1$. But $$P(S_n = 0) \ge P(X_j = 0 \text{ for all }j) > 1 - \sum_{j=1}^\infty 2^{-2j} = 2/3$$